[obm-l] Re: [obm-l] Re: [obm-l] Re: [obm-l] dúvida

2015-07-09 Por tôpico Israel Meireles Chrisostomo
Obrigado Ralph Em 9 de julho de 2015 12:37, Ralph Teixeira ralp...@gmail.com escreveu: Vamos generalizar para R^n: com a noção usual (Euclideana) de comprimento, o comprimento do segmento que liga (x1,x2,...,xn) a (y1,y2,...,yn) é: d=raiz((y1-x1)^2+(y2-x2)^2+...+(yn-xn)^2) Esta é a noção

[obm-l] Dúvida sistemas lineares

2015-07-08 Por tôpico Israel Meireles Chrisostomo
Dados a,b e c reais quaisquer sempre existem x,y e z tais que a=y+z, b=x+z e c=x+y? -- Esta mensagem foi verificada pelo sistema de antiv�rus e acredita-se estar livre de perigo.

[obm-l] dúvida

2015-07-08 Por tôpico Israel Meireles Chrisostomo
Como posso encontrar o comprimento de um segmento de reta no espaço tridimensional?Considere a origem da reta no ponto (x_0,y_0,z_0) e o final da reta no ponto (x_1,y_1,z_1) -- Esta mensagem foi verificada pelo sistema de antiv�rus e acredita-se estar livre de perigo.

[obm-l] Dúvida Desigualdades

2015-06-16 Por tôpico Israel Meireles Chrisostomo
Sejam a,b e c números reais quaisquer , então, sempre existe números x,y e z tais que a=yz/x,b=xz/y e c=xy/z? -- Esta mensagem foi verificada pelo sistema de antiv�rus e acredita-se estar livre de perigo.

[obm-l] Dúvida 'boba' de contagem

2015-06-04 Por tôpico Eduardo Henrique
E ai pessoas, tudo bem? Tava resolvendo uns exercícios sobre contagem duma apostila aqui e me deparei com este exercício: Num Ka Kay, o oriental famoso por sua inabalável paciência, deseja bater o recorde mundial de construção de castelo de cartas.Ele vai montar um castelo na forma de um prisma

Re: [obm-l] Re: [obm-l] Dúvida

2015-06-02 Por tôpico André Chaves
Ralph, Como sempre brilhante! Muito obrigado! Abração, André. Enviado do meu iPhone Em 01/06/2015, às 20:12, Ralph Teixeira ralp...@gmail.com escreveu: Bom, depende muito do que chamamos de formas... Vou supor que as posicoes sao todas importantes e rotuladas. Digo, vou contar como

[obm-l] Dúvida

2015-06-01 Por tôpico André Chaves
Prezados, Alguém pode me ajudar com a solução deste problema. Ele consta de uma lista de exercícios que apresenta a resposta 144. Um aluno meu achou 192 e eu achei 288. De antemão, muito obrigado! Abração, André Luiz. A figura abaixo é composta de 16 quadrados menores. De quantas formas é

[obm-l] Re: [obm-l] Dúvida

2015-06-01 Por tôpico Ralph Teixeira
Bom, depende muito do que chamamos de formas... Vou supor que as posicoes sao todas importantes e rotuladas. Digo, vou contar como *diferentes* preenchimentos que difiram por rotacao, reflexao ou permutacao dos numeros. Eu comecaria notando que se alguma posicao preenche o quadrado, entao

[obm-l] Dúvida

2015-05-30 Por tôpico Andre Chaves (Andrezinho)
A figura abaixo é composta de 16 quadrados menores. De quantas formas é possível preencher estes quadrados com os números 1, 2, 3 e 4, de modo que um número não possa aparecer 2 vezes em: • uma mesma linha. • uma mesma coluna. • cada um dos quatro quadrados demarcados pelas linhas contínuas.

[obm-l] Dúvida Análise:séries formais

2015-05-06 Por tôpico Israel Meireles Chrisostomo
Se eu tiver uma série dupla do tipo ΣΣa_k(n), então, eu posso aplicar a ideia de convergência uniforme para dizer que o limite da soma é a soma dos limites, isto é, passar o limite para dentro do somatório?No caso, eu poderia dizer que se |a_k(n)|M_k e se ΣM_k converge, então concluir, pelo

Re: [obm-l] Dúvida Análise(tannery's theorem)

2015-05-05 Por tôpico Artur Costa Steiner
Não sei se entendi bem sua dúvida. Mas veja que uma série é a sequência das somas parciais de uma outra sequência. Assim, se (a_n) é uma sequência de, digamos reais, então a sequência (S_n) definida por S_n = Soma (k = 1, n) a_k é a série associada a (a_n). Cada S_n é a soma dos n primeiros

Re: [obm-l] Dúvida Análise(tannery's theorem)

2015-05-05 Por tôpico Artur Costa Steiner
Não sei se entendi bem sua dúvida. Mas veja que uma série é a sequência das somas parciais de uma outra sequência. Assim, se (a_n) é uma sequência de, digamos reais, então a sequência (S_n) definida por S_n = Soma (k = 1, n) a_k é a série associada a (a_n). Cada S_n é a soma dos n primeiros

Re: [obm-l] Dúvida Análise

2015-05-05 Por tôpico Artur Costa Steiner
Vc está se referindo a séries de funções? Se estiver, a resposta de modo geral é não. O que garante a igualdade é convergência uniforme. Artur Costa Steiner Em 05/05/2015, às 03:43, Israel Meireles Chrisostomo israelmchrisost...@gmail.com escreveu: Em toda série convergente o limite da

[obm-l] Re: [obm-l] Re: [obm-l] Dúvida Análise(tannery's theorem)

2015-05-05 Por tôpico Israel Meireles Chrisostomo
Obrigado a ambos, as suas respostas são ambas interessantes.Em particular quero agradecer ao Ralph, que mesmo depois de eu o contrapor em argumentos(que por sinal eram infundados) em uma outa pergunta, mesmo assim respondeu com paciência minha dúvida Em 5 de maio de 2015 10:40, Ralph Teixeira

[obm-l] Re: [obm-l] Dúvida Análise

2015-05-05 Por tôpico Israel Meireles Chrisostomo
Obrigado, Artur Costa Steiner Em 5 de maio de 2015 09:45, Artur Costa Steiner steinerar...@gmail.com escreveu: Vc está se referindo a séries de funções? Se estiver, a resposta de modo geral é não. O que garante a igualdade é convergência uniforme. Artur Costa Steiner Em 05/05/2015, às

[obm-l] Re: [obm-l] Dúvida Análise(tannery's theorem)

2015-05-05 Por tôpico Ralph Teixeira
Não sei se entendi a pergunta também... Mas *talvez* esse seja um exemplo bom... Considere a sequencia dupla a(k,n) (onde k,n=1,2,3,...) dada por: a(k,n) = 1/k se n=k a(k,n) = 0 se nk Ou seja, mais explicitamente, colocando k fixo e variando n em cada linha: a(1,n): 1,0,0,0,0,0,0,.. a(2,n):

[obm-l] Dúvida Análise

2015-05-05 Por tôpico Israel Meireles Chrisostomo
Em toda série convergente o limite da soma é a soma dos limtes?Se isso for verdade alguém tem a demonstração? -- Esta mensagem foi verificada pelo sistema de antiv�rus e acredita-se estar livre de perigo.

Re: Re: [obm-l] Dúvida sobre Álgebra

2015-05-04 Por tôpico Listeiro 037
Olá! Obrigado por retornar! Estive pesquisando neste meio-tempo sobre avanços na teoria dos números de Fermat. Cheguei a uma proposição sobre que seguiriam um padrão de 'escada' de potências de 2: F(0) = 2 + 1 = 3 F(1) = 2^{2} + 1 = 5 F(2) = 2^{2^{2}} + 1 = 2^4 + 1 = 17 F(4) = 2^{2^{2^{2}}} +

[obm-l] Dúvida Análise(tannery's theorem)

2015-05-04 Por tôpico Israel Meireles Chrisostomo
Olá tenho um dúvida de análise seja a_k(n) um termo dependente de n e a_k o resultado do limite lim n-inf a_k(n)=a_k, se |Sa_k(n)-Sa_k|épsilon, com épsilon maior que zero então, isto significa dizer que lim n-inf Sa_k(n)=S a_k(em que S está no lugar de sigma e representa a soma da série)?Se a

Re: [obm-l] Dúvida sobre Álgebra

2015-04-13 Por tôpico gugu
Saudações. A sua afirmação é equivalente a dizer que 3, 5, 17, 257 e 65537 são os únicos primos de Fermat (o que está em aberto, e muitos matemáticos consideram provável). Se F_n=2^{2^n}+1, F_n-2=2^{2^n}-1 é o produto dos F_k de k=0 até n-1 (por exemplo, 255=3*5*17), o que pode ser

[obm-l] Dúvida sobre Álgebra

2015-04-06 Por tôpico Listeiro 037
Saudações. Tenho a dúvida sobre como se pode demonstrar (se for realmente verdade) que se 'p' é primo e divide uma circunferência com instrumentos euclidianos, então p-1 e p-2 também a divide. Ou seja, se existirem infinitos pp então existem infinitas tríades de consecutivos. Na verdade p tem

[obm-l] Re: [obm-l] Dúvida

2015-03-30 Por tôpico Douglas Oliveira de Lima
Livro do José Plínio dos Santos é bem didático. Abraço, Douglas oliveira Em 30/03/2015 13:01, Israel Meireles Chrisostomo israelmchrisost...@gmail.com escreveu: Alguém sabe um material mais completo do que o do Eduardo tengan(revista eureka n 11) em português falando sobre funções geradoras?

[obm-l] Dúvida

2015-03-30 Por tôpico Israel Meireles Chrisostomo
Alguém sabe um material mais completo do que o do Eduardo tengan(revista eureka n 11) em português falando sobre funções geradoras? -- Esta mensagem foi verificada pelo sistema de antiv�rus e acredita-se estar livre de perigo.

[obm-l] Dúvida

2015-03-29 Por tôpico Andre Chaves (Andrezinho)
Prezados, Em anexo há uma planilha do Excel com dados colhidos a partir de um jogo. Peço a ajuda de vocês para obter um polinômio ou a descrição de um método ou indicação de um software que aproxima os dados apresentados. Agradecendo desde já, despeço-me. Atenciosamente, André Chaves --- Este

[obm-l] Re: [obm-l] Dúvida

2015-03-29 Por tôpico Ralph Teixeira
O proprio Excel tem algo que pode ajudar. Tente isso: 1. Fazer o grafico Scatterplot XY dos seus dados 2. Clicar em um dos pontos que voce plotou, e selecione Adicionar Linha de Tendencia. Voce tem que escolher o modelo (linear, exponencial), mas ele faz o resto. Se voce pedir, ele te mostra a

Re: [obm-l] Re: [obm-l] Dúvida

2015-03-29 Por tôpico Andrezinho
Muito obrigado, Ralph. Obviamente seguirei seu conselho. Abração. Enviado do meu iPhone Em 29/03/2015, às 16:00, Ralph Teixeira ralp...@gmail.com escreveu: O proprio Excel tem algo que pode ajudar. Tente isso: 1. Fazer o grafico Scatterplot XY dos seus dados 2. Clicar em um dos pontos

Re: [obm-l] Dúvida

2015-03-07 Por tôpico nepier
Em 05/03/2015 21:41, Cláudio Thor escreveu: NUMA CERTA TURMA, PRATICAM ALGUM ESPORTE, EXATAMENTE, 3 EM CADA 4 RAPAZES E, TAMBÉM, 95% DAS MENINAS. NO TOTAL ISSO REPRESENTA 80 % DAS PESSOAS DESTA TURMA. QUAL É A PORCENTAGEM DE MOÇAS NESTA TURMA? 75% R+ 95%M = 80%(R+M) 25%R+5%M =

[obm-l] Dúvida

2015-03-05 Por tôpico Cláudio Thor
Numa certa turma, praticam algum esporte, exatamente, 3 em cada 4 rapazes e, também, 95% das meninas. No total isso representa 80 % das pessoas desta turma. Qual é a porcentagem de moças nesta turma? Alguém tem uma solução rápida para esta questão. Agradeço From: zitinho...@hotmail.com To:

[obm-l] RE: [obm-l] Dúvida

2015-03-05 Por tôpico Eduardo Henrique
A menos de contas erradas, creio dar 25% de moças da turma. Utilizando o procedimento que eu sugeri ali no outro email, R=1-M = 0,95M+0,75(1-M)=0,8 = 0,95M - 0,75M +0,75 = 0,8 = = 0,2M=0,05 = M=0,25 = 25%. From: dr.dhe...@outlook.com To: obm-l@mat.puc-rio.br Subject: RE: [obm-l] Dúvida Date

[obm-l] RE: [obm-l] Dúvida

2015-03-05 Por tôpico Eduardo Henrique
Rapazes+Moças = R+M = 1 = 100% 3R/4 + 95M/100 = 0,8 Sistema de duas eq. e duas incógnitas. Só resolver isolando R na primeira eq e substituindo na segunda. Att. Eduardo From: claudiot...@hotmail.com To: obm-l@mat.puc-rio.br Subject: [obm-l] Dúvida Date: Fri, 6 Mar 2015 00:41:50 + Numa

[obm-l] Dúvida Combinatória

2014-09-29 Por tôpico Jorge Paulino
Num grupo de 11 pessoas, 2 são brasileiros, 5 são argentinos, 3 são franceses e 1 é português. Quantas permutações podemos formar com essas 11 pessoas, de modo que não haja brasileiro ao lado de argentino? Grato, Jorge -- Esta mensagem foi verificada pelo sistema de antivírus e acredita-se

[obm-l] Re: [obm-l] Dúvida Combinatória

2014-09-29 Por tôpico Mauricio de Araujo
Olá Jorge!! vou dar apenas uma indicação de como acho que daria para chegar numa resposta... Observe a figura abaixo: _U_U_U_U_ Coloquemos nas posições U os 3 franceses e o portugues. Temos 4! de possibilidades para fazer isso. Agora precisamos colocar os brasileiros na posições _, podendo

[obm-l] Re: [obm-l] Dúvida Combinatória

2014-09-29 Por tôpico Mauricio de Araujo
desculpe não tem erro algum... desconsidere o email imediatamente acima... 2014-09-29 22:02 GMT-03:00 Mauricio de Araujo mauricio.de.ara...@gmail.com : tem um erro na maneira como abri os casos... descubra qual é... 2014-09-29 21:54 GMT-03:00 Mauricio de Araujo mauricio.de.ara...@gmail.com:

[obm-l] Dúvida simples!!

2014-09-01 Por tôpico Cláudio Thor
O proprietário de uma casa de espetáculos observou que, colocando o valor da entrada a R$10,00, sempre contava com 1.000 pessoas a cada apresentação, faturando R$10.000,00 com a venda dos ingressos. Entretanto, percebeu também que, a partir de R$10,00, a cada R$2,00 que ele aumentava no valor da

Re: [obm-l] Dúvida simples!!

2014-09-01 Por tôpico Artur Costa Steiner
Se o preço da entrada for p= 10, então teremos que P = 1000 - (p - 10)/2 . 40 = 1000 - 20p + 200 = 1200 - 20p Logo, p = (1200 - P)/20 = 60 - P/20, 0 = P = 1000 E o faturamento é F = p P = 60P - (P^2)/20, 0 = P = 1000 Artur Costa Steiner Em 01/09/2014, às 22:35, Cláudio Thor

[obm-l] Re: [obm-l] Dúvida em análise complexa, função Lipschitz

2014-08-10 Por tôpico Bernardo Freitas Paulo da Costa
Lembre que uma função C^1 é localmente Lipschitz. 2014-08-09 16:27 GMT-03:00 Merryl sc...@hotmail.com: Eu estou me enrolando nisso. Se f é inteira, então f é Lipschitz em todo conjunto limitado do plano complexo. Estou me enrolando para provar, podem ajudar? Obrigada -- Esta mensagem foi

[obm-l] Dúvida em análise complexa, função Lipschitz

2014-08-09 Por tôpico Merryl
Eu estou me enrolando nisso. Se f é inteira, então f é Lipschitz em todo conjunto limitado do plano complexo. Estou me enrolando para provar, podem ajudar? Obrigada -- Esta mensagem foi verificada pelo sistema de antivírus e acredita-se estar livre de

[obm-l] RE: [obm-l] Re: [obm-l] Dúvida(questão simples)

2014-02-22 Por tôpico marcone augusto araújo borges
Eu pensei assim também.Obrigado! From: ilhadepaqu...@bol.com.br To: obm-l@mat.puc-rio.br Subject: [obm-l] Re: [obm-l] Dúvida(questão simples) Date: Sat, 22 Feb 2014 00:28:27 -0300 quando ele anda no sentido horário ele anda 380 graus em 40 minutos porque o ponteiro das horas em 40

[obm-l] RE: [obm-l] Re: [obm-l] Dúvida(questão simples)

2014-02-22 Por tôpico marcone augusto araújo borges
Obrigado! Date: Sat, 22 Feb 2014 00:31:24 -0300 Subject: [obm-l] Re: [obm-l] Dúvida(questão simples) From: tarsise...@gmail.com To: obm-l@mat.puc-rio.br Erramos juntos. Pq tb achei 58. -- Esta mensagem foi verificada pelo sistema de antivírus e acredita-se estar livre de perigo

[obm-l] Dúvida(questão simples)

2014-02-21 Por tôpico marcone augusto araújo borges
Exatamente no momento em que o ponteiro das horas passa pelo 12, uma formiga começa a andar ao longo da borda de um relógio no sentido anti-horário,partindo do 6,com velocidade constante.Quando a formiga en- contra o ponteiro das horas,ela muda de direção e continua a andar na mesma velocidade

[obm-l] Re: [obm-l] Dúvida(questão simples)

2014-02-21 Por tôpico Tarsis Esau
Erramos juntos. Pq tb achei 58. -- Esta mensagem foi verificada pelo sistema de antivírus e acredita-se estar livre de perigo.

[obm-l] Re: [obm-l] Dúvida(questão simples)

2014-02-21 Por tôpico Hermann
é de 10 graus por minuto para percorrer 180 graus eles vão demorar 18 minutos 40 +18 = 58 minutos abraços Hermann - Original Message - From: marcone augusto araújo borges To: obm-l@mat.puc-rio.br Sent: Friday, February 21, 2014 11:33 PM Subject: [obm-l] Dúvida(questão

Re: [obm-l] dúvida

2012-12-15 Por tôpico Artur Costa Steiner
no de pessoas com renda maior que 10.000 = (10^12)/(10.000^2) = (10^12)/(10^8)= 10^4 = 10.000, inteiro. no de pessoas com renda maior que 19.999,99= maior inteiro (10^12)/(19.999,99)^2 Tome a diferença entre os dois valores Artur Costa Steiner Em 14/12/2012, às 22:26, JOSE AIRTON CARNEIRO

[obm-l] Re: [obm-l] dúvida

2012-12-15 Por tôpico JOSE AIRTON CARNEIRO
obrigado Artur. Em 15 de dezembro de 2012 08:19, Artur Costa Steiner steinerar...@gmail.com escreveu: no de pessoas com renda maior que 10.000 = (10^12)/(10.000^2) = (10^12)/(10^8)= 10^4 = 10.000, inteiro. no de pessoas com renda maior que 19.999,99= maior inteiro (10^12)/(19.999,99)^2

[obm-l] dúvida

2012-12-14 Por tôpico JOSE AIRTON CARNEIRO
Amigos, por favor, uma ajuda nesse problema: Numa determinada comunidade, o número de pessoas cuja renda anual excede o valor x (em reais) é igual a (10^12)/x^2. Quantas pessoas nessa comunidade têm uma renda anual entre 10.000 e 20.000? a) 10.000 b) 2.500 c) 7.500 d) 12.500

[obm-l] dúvida em teoria dos números

2012-05-17 Por tôpico Marco Antonio Leal
Provar que 10 ^11 - 1 é divisivel por 100

[obm-l] Re: [obm-l] dúvida em teoria dos números

2012-05-17 Por tôpico Bernardo Freitas Paulo da Costa
On Thu, May 17, 2012 at 12:18 PM, Marco Antonio Leal marcoantonio_elemen...@hotmail.com wrote: Provar que 10 ^11 - 1 é divisivel por 100 Um número só com noves não tem grandes chances de ser divisível por 100. Pior ainda, um número ímpar não é divisível por nenhum par. E não adianta nem tentar

[obm-l] Re: [obm-l] RE: [obm-l] RE: [obm-l] Dúvida Indução

2012-05-17 Por tôpico Ralph Teixeira
ponto, fazia 2+5+8+...+(3n-1)+[(3n-1)+1], chegando aí eu me perco From: joao_maldona...@hotmail.com To: obm-l@mat.puc-rio.br Subject: [obm-l] RE: [obm-l] Dúvida Indução Date: Mon, 14 May 2012 15:24:47 -0300 Vamos dizer que para n respeite a formula Logo 2+4+6

[obm-l] Re: [obm-l] dúvida em teoria dos números

2012-05-17 Por tôpico luiz silva
Tem certeza que o enunciado é essse ? De: Marco Antonio Leal marcoantonio_elemen...@hotmail.com Para: obm-l@mat.puc-rio.br Enviadas: Quinta-feira, 17 de Maio de 2012 7:18 Assunto: [obm-l] dúvida em teoria dos números Provar que 10 ^11 - 1 é divisivel por 100

[obm-l] RE: [obm-l] RE: [obm-l] Dúvida Indução

2012-05-16 Por tôpico Thiago Bersch
Então eu estava tentando fazer mas parava no mesmo ponto, fazia 2+5+8+...+(3n-1)+[(3n-1)+1], chegando aí eu me perco From: joao_maldona...@hotmail.com To: obm-l@mat.puc-rio.br Subject: [obm-l] RE: [obm-l] Dúvida Indução Date: Mon, 14 May 2012 15:24:47 -0300 Vamos dizer que para n respeite

[obm-l] RE: [obm-l] Dúvida Indução

2012-05-14 Por tôpico João Maldonado
From: thiago_t...@hotmail.com To: obm-l@mat.puc-rio.br Subject: [obm-l] Dúvida Indução Date: Mon, 14 May 2012 01:09:39 -0300 2 + 4 + . . . + 2n. 2 + 5 + 8 + . . . + (3n-1).Bem eu sei que o primeiro irá dar n(n+1) e o segundo n(3n+1)/2O que em si eu não entendi o resultado O primeiro eu

[obm-l] Dúvida Indução

2012-05-13 Por tôpico Thiago Bersch
2 + 4 + . . . + 2n. 2 + 5 + 8 + . . . + (3n-1).Bem eu sei que o primeiro irá dar n(n+1) e o segundo n(3n+1)/2O que em si eu não entendi o resultado O primeiro eu tentei fazer assim:2+4...+2n + n+2n+(2n+1), e fiquei parado nisso e o segunda também, gostaria de uma explicação passo-a-passo

[obm-l] dúvida em uma afirmação de um vestibular da UEM

2012-05-01 Por tôpico Vanderlei *
*Pessoal, a afirmação a seguir é verdadeira ou falsa? Penso que seja verdadeira, porém o gabarito do vestibular diz ser falsa! Preciso de ajuda!* * * *Em um plano, existem duas figuras F1 e F2, cujas bases estão sobre uma reta r do plano, com a seguinte propriedade: * *toda reta paralela à reta

[obm-l] Re: [obm-l] dúvida em uma afirmação de um vestibular da UEM

2012-05-01 Por tôpico Bernardo Freitas Paulo da Costa
2012/5/1 Vanderlei * vanderma...@gmail.com: Pessoal, a afirmação a seguir é verdadeira ou falsa? Penso que seja verdadeira, porém o gabarito do vestibular diz ser falsa! Preciso de ajuda! Em um plano, existem duas figuras F1  e F2, cujas bases estão sobre uma reta r  do plano, com a seguinte

[obm-l] RES: [obm-l] Re: [obm-l] dúvida em uma afirmação de um vestibular da UEM

2012-05-01 Por tôpico bouskela
...@mat.puc-rio.br] Em nome de Bernardo Freitas Paulo da Costa Enviada em: terça-feira, 1 de maio de 2012 08:23 Para: obm-l@mat.puc-rio.br Assunto: [obm-l] Re: [obm-l] dúvida em uma afirmação de um vestibular da UEM 2012/5/1 Vanderlei * vanderma...@gmail.com: Pessoal, a afirmação a seguir é verdadeira

[obm-l] Re: [obm-l] Re: [obm-l] Dúvida de geometria analítica: É perpendicular, portanto... (?)

2012-03-22 Por tôpico Felipe Blassioli
perpendiculares, daí segue a condição de que o produto escalar a(x_1 - x_2) + b (y_1 - y_2) seja nulo. [ ]'s --- Em *qua, 21/3/12, Felipe Blassioli felipeblassi...@gmail.com*escreveu: De: Felipe Blassioli felipeblassi...@gmail.com Assunto: [obm-l] Dúvida de geometria analítica: É

[obm-l] Dúvida

2012-03-12 Por tôpico Vanessa Nunes de Souza
Gostaria da ajuda dos colegas para resolver duas questões:

[obm-l] Re: [obm-l] Dúvida

2012-03-12 Por tôpico tarsis Esau
a) Fazendo-se a diferença entre (100c+10b+a) - (100a+10b+c) = 396 99c-99a=396 c-a=4 Sendo 2c=a = a=4 e c=8, como a,b,c formando uma PA, temos que b=6 b) A área do triângulo QMN pode ser dada por S1 = [(QP/2)x(QR/2)xsenQ]/2 = S1=(QPxQRxsenQ)/8 A área do triângulo QPR pode ser dada por S2 =

[obm-l] RE: [obm-l] Re: [obm-l] Dúvida

2012-03-12 Por tôpico Vanessa Nunes de Souza
Obrigada!Vanessa Nunes Date: Mon, 12 Mar 2012 17:04:46 -0300 Subject: [obm-l] Re: [obm-l] Dúvida From: tarsise...@gmail.com To: obm-l@mat.puc-rio.br a) Fazendo-se a diferença entre (100c+10b+a) - (100a+10b+c) = 396 99c-99a=396 c-a=4 Sendo 2c=a = a=4 e c=8, como a,b,c formando uma PA, temos

[obm-l] Re: [obm-l] Re: [obm-l] Dúvida de Lógica

2012-02-08 Por tôpico Ralph Teixeira
H nao sei nao. Vou usar C para estah contido e E para pertence a. Concordo que toda inclusao de conjuntos pode ser pensada como uma implicacao (bom, com um quantificador para todo). Afinal: A C B eh o mesmo que dizer para todo x, xEA = xEB Por isso, concordo que a Teoria dos Conjuntos e

[obm-l] Dúvida de Lógica

2012-02-07 Por tôpico Pedro Chaves
Caros Colegas, Pode-se dizer que 2 3 = Todo brasileiro é desonesto?   (O símbolo = indica implicação lógica.) Sei que é verdadeira a proposição condicional Se 2 3, então todo brasileiro é desonesto, mas me parece que não existe implicação lógica. Desde já, muito obrigado. Um abraço do

[obm-l] Re: [obm-l] Dúvida de Lógica

2012-02-07 Por tôpico Francisco Barreto
Saudações a todos! Seja A o conjunto dos objetos que satisfazem a propriedade r de que 2 3. Seja B o conjunto dos objetos que satisfazem a propriedade s de que Todo brasileiro é desonesto Sabemos que A é o conjunto vazio. O conjunto vazio está contido em qualquer conjunto, incluindo B. Portanto

[obm-l] Re: [obm-l] Dúvida de Lógica

2012-02-07 Por tôpico Ralph Teixeira
Sim, **logicamente**, a frase 23 == Eu sou o papa estah correta. A implicacao logica eh um simbolo DEFINIDO por esta tabela-verdade: p q p = q V V V V F F F V V F F V e, a principio, eh soh isso. Nada em p=q **intrinsicamente** significa causa, efeito, razao ou qualquer coisa

[obm-l] Re: [obm-l] Dúvida de Lógica

2012-02-07 Por tôpico Francisco Barreto
o termo = traduz-se em se-então. Se todos os elementos de A satisfazem r então todos os elementos de A satisfazem s. Que zona! 2012/2/7 Francisco Barreto fcostabarr...@gmail.com Saudações a todos! Seja A o conjunto dos objetos que satisfazem a propriedade r de que 2 3. Seja B o conjunto

[obm-l] Dúvida em gabarito

2011-11-26 Por tôpico marcone augusto araújo borges
seja um quadrado ABCD de lado a.Os arcos AC e BD formam com 3 dos lados do quadrado dois setores ciculares(um quarto do círculo de raio a cada).Deteminar a área comum aos setores circulares. A resposta que o livro dá é a^2(9raiz(3) - 4pi)/12 e a que eu achei foi a^2(4pi - 3raiz(3))/12 Dsculpem

[obm-l] Dúvida

2011-11-02 Por tôpico Kleber Bastos
Olá grupo, Estou me enrolando nesta prova. Mostre q ∀ nº a/b0, MDC(a,b) = 1, é válido: f(a/b) = f(1)^a/b . -- Kleber.

[obm-l] Re: [obm-l] Dúvida

2011-11-02 Por tôpico Victor Hugo Rodrigues
Como assim? Acho que falta algo aí. Em 2 de novembro de 2011 17:17, Kleber Bastos klebe...@gmail.com escreveu: Olá grupo, Estou me enrolando nesta prova. Mostre q ∀ nº a/b0, MDC(a,b) = 1, é válido: f(a/b) = f(1)^a/b . -- Kleber.

[obm-l] Re: [obm-l] Re: [obm-l] Dúvida

2011-11-02 Por tôpico Kleber Bastos
É isso mesmo: Mostrar que ∀ nº racional a/b0, M.D.C.(a,b)=1 é válida a sentença: f(a/b)=f(1)^a/b ( f(1) elevado a a/b) Em 2 de novembro de 2011 20:57, Victor Hugo Rodrigues victorhcr.victorh...@gmail.com escreveu: Como assim? Acho que falta algo aí. Em 2 de novembro de 2011 17:17, Kleber

[obm-l] Re: [obm-l] Re: [obm-l] Re: [obm-l] Dúvida

2011-11-02 Por tôpico Joao Maldonado
-feira, 2 de Novembro de 2011 22:21 Assunto: [obm-l] Re: [obm-l] Re: [obm-l] Dúvida É isso mesmo: Mostrar que ∀ nº racional a/b0, M.D.C.(a,b)=1 é válida a sentença: f(a/b)=f(1)^a/b ( f(1) elevado a a/b) Em 2 de novembro de 2011 20:57, Victor Hugo Rodrigues victorhcr.victorh...@gmail.com escreveu

[obm-l] Re: [obm-l] Re: [obm-l] Re: [obm-l] Re: [obm-l] Dúvida

2011-11-02 Por tôpico Kleber Bastos
consegui pensar até agora é f(x) = a^x f(x) = x², f(4/7) = 16/49 != 1^(4/7) = 1 []'s João -- Date: Wed, 2 Nov 2011 17:17:59 -0200 Subject: [obm-l] Dúvida From: klebe...@gmail.com To: obm-l@mat.puc-rio.br Olá grupo, Estou me enrolando nesta prova. Mostre q ∀ nº a/b0

[obm-l] Dúvida

2011-10-24 Por tôpico Jorge Paulino da Silva Filho
Dois amigos, Lucas e Pedro, seguiam o leito de uma ferrovia e começaram a atravessar uma ponte estreita na qual havia espaço apenas para o trem. No momento em que completavam 2/5 do percurso da ponte, ouviram o trem que se aproxima por trás deles. Lucas começou a correr de encontro ao trem,

[obm-l] Re: [obm-l] Dúvida

2011-10-24 Por tôpico Hugo Fernando Marques Fernandes
Lucas corre 2/5 da ponte com velocidade de 15 km/h. Sendo p o comprimento da ponte, leva (2/5)p/15 = 2p/75 h para sair da ponte. Pedro corre 3/5 da ponte com velocidade de 15 km/h. Sendo p o comprimento da ponte, leva (3/5)p/15 = 3p/75 h para sair da ponte. A diferença entre o momento em que

[obm-l] Re: [obm-l] Re: [obm-l] Dúvida

2011-10-24 Por tôpico Julio César Saldaña
pedro, portanto su velocidade é 5x15=75. Julio Saldaña -- Mensaje original --- De : obm-l@mat.puc-rio.br Para : obm-l@mat.puc-rio.br Fecha : Mon, 24 Oct 2011 17:08:45 -0200 Asunto : [obm-l] Re: [obm-l] Dúvida Lucas corre 2/5 da ponte com velocidade de 15 km/h. Sendo p o comprimento da ponte

[obm-l] RE: [obm-l] Re: [obm-l] dúvida - teoria dos números

2011-08-05 Por tôpico Rhilbert Rivera
16:00:56 -0700 From: jeffma...@yahoo.com.br Subject: [obm-l] Re: [obm-l] dúvida - teoria dos números To: obm-l@mat.puc-rio.br Tente pensar no pequeno Teorema de Newton, ou se preferir use congruencias.abs De: Marco Antonio Leal marcoantonio_elemen...@hotmail.com Para: obm-l@mat.puc-rio.br Enviadas

[obm-l] Re: [obm-l] dúvida - teoria dos números

2011-07-31 Por tôpico Jefferson Franca
Tente pensar no pequeno Teorema de Newton, ou se preferir use congruencias. abs De: Marco Antonio Leal marcoantonio_elemen...@hotmail.com Para: obm-l@mat.puc-rio.br Enviadas: Quinta-feira, 28 de Julho de 2011 21:39 Assunto: [obm-l] dúvida - teoria dos números

[obm-l] Re: [obm-l] dúvida - teoria dos números

2011-07-31 Por tôpico Jefferson Franca
21:39 Assunto: [obm-l] dúvida - teoria dos números Não estou conseguindo uma prova satisfatória para o seguinte exercício:   prove que 2 ^23  -  1 é divisivel por 47.

[obm-l] dúvida - teoria dos números

2011-07-28 Por tôpico Marco Antonio Leal
Não estou conseguindo uma prova satisfatória para o seguinte exercício: prove que 2 ^23 - 1 é divisivel por 47.

[obm-l] RE: [obm-l] dúvida - teoria dos números

2011-07-28 Por tôpico João Maldonado
primo, ou 2^23-1 ou 2^23+1 faz a divisãoMas ainda não prova nada []'sJoão From: marcoantonio_elemen...@hotmail.com To: obm-l@mat.puc-rio.br Subject: [obm-l] dúvida - teoria dos números Date: Fri, 29 Jul 2011 03:39:45 +0300 Não estou conseguindo uma prova satisfatória para o seguinte

[obm-l] Res: [obm-l] Enc: Re: [obm-l] FW: [obm-l] Dúvida em Geometria

2011-06-15 Por tôpico Paulo Barclay Ribeiro
Enviadas: Sexta-feira, 10 de Junho de 2011 0:25:27 Assunto: [obm-l] Enc: Re: [obm-l] FW: [obm-l] Dúvida em Geometria Estou repetindo a mensagem pois o que apareceu na lista está muito deformado em relação ao que eue enviei antes; os simbolos vetoriais devem estar em negrito, que talvez o copilador

[obm-l] Re: [obm-l] FW: [obm-l] Dúvida em Geometria

2011-06-09 Por tôpico Eduardo Wilner
diedro = - n1 . n2 = -(32*32)/(16*16*5) = - 4/5. [ ]'s --- Em dom, 29/5/11, João Maldonado joao_maldona...@hotmail.com escreveu: De: João Maldonado joao_maldona...@hotmail.com Assunto: [obm-l] FW: [obm-l] Dúvida em Geometria Para: obm-l@mat.puc-rio.br Data: Domingo, 29 de Maio de

[obm-l] Enc: Re: [obm-l] FW: [obm-l] Dúvida em Geometria

2011-06-09 Por tôpico Eduardo Wilner
...@yahoo.com.br escreveu: De: Eduardo Wilner eduardowil...@yahoo.com.br Assunto: Re: [obm-l] FW: [obm-l] Dúvida em Geometria Para: obm-l@mat.puc-rio.br Data: Quinta-feira, 9 de Junho de 2011, 17:01 É uma boa oportunidade de aplicar vetores; o produto escalar dos versores das normais às faces fornece o

[obm-l] Re: [obm-l] Re: [obm-l] RE: [obm-l] Re: [obm-l] Re: [obm-l] dúvida sobre séries

2011-06-08 Por tôpico Artur Costa Steiner
O critério mais simples para mostrar que a série harmônica diverge talvez seja o baseado no seguinte teorema: Se x_n é uma sequência decrescente de reais tal que Soma x_n converge, então lim n x_ n = 0. (Prove isto) Se x_n = 1/n, x_n decresce para 0 mas lim n x_n = 1, o que mostra que Soma x_n

[obm-l] RE: [obm-l] Re: [obm-l] Re: [obm-l] dúvida sobre séries

2011-06-07 Por tôpico Luís Lopes
Sauda,c~oes, Legal este critério, parece ter sido criado para a série harm. E a esse respeito, o autor da pergunta poderia ler também sobre a constante de Euler. []'s Luís Date: Mon, 6 Jun 2011 23:50:37 -0300 Subject: [obm-l] Re: [obm-l] Re: [obm-l] dúvida sobre séries From

[obm-l] Re: [obm-l] RE: [obm-l] Re: [obm-l] Re: [obm-l] dúvida sobre séries

2011-06-07 Por tôpico Rodrigo Renji
Olá! Então acho bem bacana esse também ( e nem é tão complicado de demonstrar, eu acho ) Esse critério pode ser usado para estudar a convergência de [ SOMA de 1/ k^p ] também pois [ SOMA de 2^k / 2^(kp) ] = [ SOMA de 2^(k (1-p)) ] se 1 - p 0, isto é 1 p a série converge por série

[obm-l] dúvida sobre séries

2011-06-06 Por tôpico claudinei
Pessoal definitivamente nao consigo entender pq a série [somatória de 1/n] com n indo de 1 ao infinito, divergepq nao converge para zero alguém por favor poderia me explicar???

[obm-l] Re: [obm-l] dúvida sobre séries

2011-06-06 Por tôpico Ralph Teixeira
Cuidado: nao confunda o TERMO GERAL de uma serie com a SERIE em si... Na serie SOMATORIO(a_n), o termo geral eh a_n. Mas a serie consiste em SOMAR todos esses a_n. A SEQUENCIA 1/n converge para 0 quando n vai para infinito. 1/n eh o termo geral da serie SOMATORIO(1/n) -- mas nao eh a SERIE. A

[obm-l] Re: [obm-l] dúvida sobre séries

2011-06-06 Por tôpico Bruno França dos Reis
Bom, primeiro vamos deixar claro que é absolutamente impossível que ela convirja para 0. Seja a_n a sequência definida por a_n = 1/n, para todo n = 1. Seja s_n a n-ésima soma parcial da série, isto é, s_n = soma[i = 1 .. n] a_i = soma[i = 1 .. n] 1/i. A soma da sua série é igual a lim[n -- +oo]

[obm-l] Re: [obm-l] dúvida sobre séries

2011-06-06 Por tôpico Victor Seixas Souza
Essa série é a série Harmônica, ela diverge porque a *soma* dos seus termos vai para o infinito. Mais tecnicamente, a soma dos termos pode ficar tão grande quanto se queira aumentando a quantidade de termos. Existe uma prova clássica para iss, feita pelo Nicolau d'Oresme e é a seguinte: S = 1 +

[obm-l] Re: [obm-l] dúvida sobre séries

2011-06-06 Por tôpico Emanuel Valente
Vamo lá... acho q aqui vai ser mais fácil entendre... Desenhe os eixos x e y e vários retangulos juntos com base 1 e de aréa 1, 1/2, 1/4 +... trace a curva 1/x nesse gráfico... vc terá a seguinte relação: Sn = 1 + 1/2 + 1/3 + ... = integral((1/x) dx), de 1 até n+1 ou seja, a soma das áreas dos

[obm-l] Re: [obm-l] Re: [obm-l] dúvida sobre séries

2011-06-06 Por tôpico Rodrigo Renji
Olá! Uma outra maneira ( além da que os colegas enviaram antes), para mostrar que a série não converge, tem um critério de convergência que acho legal, Critério de condensação de Cauchy: Se x_k é uma sequência decrescente de termos positivos ( como é o caso de 1/k ) então a série [ SOMA de

[obm-l] RE: [obm-l] Dúvida em Geometria

2011-06-02 Por tôpico João Maldonado
modo calculemos z2P =8sqrt(5)/3 = z4P. Do triângulo z2Pz4, vemos que cos o angulo desejado é -4/5 []'sJoão Date: Sat, 28 May 2011 15:37:19 -0700 From: paulobarc...@yahoo.com.br Subject: [obm-l] Dúvida em Geometria To: obm-l@mat.puc-rio.br prezados, Desculpe a dúvida, mas estou encontrando

[obm-l] Res: [obm-l] FW: [obm-l] Dúvida em Geometria

2011-06-02 Por tôpico Paulo Barclay Ribeiro
 pitágoras z5M = 2 sqrt(5), logo  sen(x) = sqrt(5)/3. Deste  modo calculemos z2P =8sqrt(5)/3 = z4P. Do triângulo z2Pz4, vemos que cos o angulo  desejado é  -4/5 []'s João Date: Sat, 28 May 2011 15:37:19 -0700 From: paulobarc...@yahoo.com.br Subject: [obm-l] Dúvida

[obm-l] FW: [obm-l] Dúvida em Geometria

2011-05-29 Por tôpico João Maldonado
From: joao_maldona...@hotmail.com To: obm-l@mat.puc-rio.br Subject: RE: [obm-l] Dúvida em Geometria Date: Sun, 29 May 2011 01:19:37 -0300 Bom, sou estudante de ensino medio, logo minha resposta pode estar errada :D Fazendo z1, z2, z3, z4 como os vértices da base e z5 como o vértice

[obm-l] Dúvida em Geometria

2011-05-28 Por tôpico Paulo Barclay Ribeiro
prezados,   Desculpe a dúvida, mas estou encontrando dificuldade num problema bem elementar, e peço uma orientação , é o seguinte:   Qual o cosseno do ângulo diedro entre duas faces de uma piramide quadrangular de altura dois e aresta da base igual a 8..Desde já agradeço.   Paulo

[obm-l] Re: [obm-l] Re: [obm-l] Dúvida - OBM Nível Universitário

2011-01-24 Por tôpico Hugo Fernando Marques Fernandes
Oi, Bruna. Pois é, eu já tinha ouvido dizer isso e queria confirmar. É uma pena, mas fazer o que? Regras são regras... Talvez fosse o caso de criar uma categoria nova pra quem já tem diploma, né? Obrigado pela resposta. Hugo. Em 24 de janeiro de 2011 02:20, Bruna Campos

[obm-l] Re: [obm-l] Re: [obm-l] Re: [obm-l] Dúvida - OBM Nível Universitário

2011-01-24 Por tôpico Tiago
Seria uma olimpiada mais dificil ainda de ganhar, haha. 2011/1/24 Hugo Fernando Marques Fernandes hfernande...@gmail.com Oi, Bruna. Pois é, eu já tinha ouvido dizer isso e queria confirmar. É uma pena, mas fazer o que? Regras são regras... Talvez fosse o caso de criar uma categoria nova

[obm-l] Re: [obm-l] Dúvida - OBM Nível Universitário

2011-01-23 Por tôpico Bruna Campos
Hugo, que eu saiba não pode. Só pode participar quem não tem diploma de curso superior :( Em 20/01/11, Hugo Fernando Marques Fernandeshfernande...@gmail.com escreveu: Boa noite. Acabo de ser aprovado para o curso de matemática da UERJ e tenho uma dúvida em relação à OBM de nível

[obm-l] Re: [obm-l] Dúvida - OBM Nível Universitário

2011-01-23 Por tôpico Bruna Campos
PS.: E só até o quarto ano de graduação. Abraços! Em 23/01/11, Bruna Camposbda.cam...@gmail.com escreveu: Hugo, que eu saiba não pode. Só pode participar quem não tem diploma de curso superior :( Em 20/01/11, Hugo Fernando Marques Fernandeshfernande...@gmail.com escreveu: Boa noite.

[obm-l] RE: [obm-l] Re: [obm -l] Dúvida-Geometria ana lítica

2010-10-11 Por tôpico marcone augusto araújo borges
diferente tambem. Obrigado pela sua atenção. Date: Sun, 10 Oct 2010 16:57:11 -0700 From: eduardowil...@yahoo.com.br Subject: [obm-l] Re: [obm-l] Dúvida-Geometria ana lítica To: obm-l@mat.puc-rio.br Pelo menos na geometria euclidiana esse triângulo inexiste: a altura relativa à AB vale 6, logo

[obm-l] Re:[obm-l] Dúvida-Geometria ana lítica

2010-10-11 Por tôpico Eduardo Wilner
Claro! Desculpe a distração anterior. Vc. pode considerar que a mediana só pode medir 3 se for perpendicular à AB. Assim, C será a intersecção do prolongamento de BD com a  paralela à AB (eixo dos x) à uma altura 6, sendo A, do triângulo original (o  que vc. mencionou como reto è o

<    1   2   3   4   5   6   7   8   9   10   >